Does series $4^{n}×frac{(n!)^2}{(2n)!}$ converge or diverge?











up vote
1
down vote

favorite
1












I know $4^{n}$ diverges, and $frac{(n!)^2}{(2n)!}$ converges. I also think that their product diverges since the term of this series is increasing but I don't know how to prove it. I tried ratio test which is inconclusive, and some other tests.










share|cite|improve this question






















  • Use Stirling's approximation for the factorial term
    – asdf
    Apr 15 at 16:33






  • 3




    It has been proved multiple times on MSE that $$frac{1}{4^n}binom{2n}{n}simfrac{1}{sqrt{pi n}},$$ so... (you do not need Stirling's inequality/approximation to prove it)
    – Jack D'Aurizio
    Apr 15 at 16:33












  • Wait - do you really ask for the series $sum 4^n/{2nchoose n}$ or just for the sequence ${4^n/{2nchoose n}}_{n=0}^infty$?
    – Hagen von Eitzen
    Apr 15 at 16:43












  • I am asking for series.
    – Nebeski
    Apr 15 at 16:47












  • More methods at math.stackexchange.com/questions/1606836/…
    – BAYMAX
    Apr 15 at 17:17















up vote
1
down vote

favorite
1












I know $4^{n}$ diverges, and $frac{(n!)^2}{(2n)!}$ converges. I also think that their product diverges since the term of this series is increasing but I don't know how to prove it. I tried ratio test which is inconclusive, and some other tests.










share|cite|improve this question






















  • Use Stirling's approximation for the factorial term
    – asdf
    Apr 15 at 16:33






  • 3




    It has been proved multiple times on MSE that $$frac{1}{4^n}binom{2n}{n}simfrac{1}{sqrt{pi n}},$$ so... (you do not need Stirling's inequality/approximation to prove it)
    – Jack D'Aurizio
    Apr 15 at 16:33












  • Wait - do you really ask for the series $sum 4^n/{2nchoose n}$ or just for the sequence ${4^n/{2nchoose n}}_{n=0}^infty$?
    – Hagen von Eitzen
    Apr 15 at 16:43












  • I am asking for series.
    – Nebeski
    Apr 15 at 16:47












  • More methods at math.stackexchange.com/questions/1606836/…
    – BAYMAX
    Apr 15 at 17:17













up vote
1
down vote

favorite
1









up vote
1
down vote

favorite
1






1





I know $4^{n}$ diverges, and $frac{(n!)^2}{(2n)!}$ converges. I also think that their product diverges since the term of this series is increasing but I don't know how to prove it. I tried ratio test which is inconclusive, and some other tests.










share|cite|improve this question













I know $4^{n}$ diverges, and $frac{(n!)^2}{(2n)!}$ converges. I also think that their product diverges since the term of this series is increasing but I don't know how to prove it. I tried ratio test which is inconclusive, and some other tests.







sequences-and-series divergent-series






share|cite|improve this question













share|cite|improve this question











share|cite|improve this question




share|cite|improve this question










asked Apr 15 at 16:32









Nebeski

21129




21129












  • Use Stirling's approximation for the factorial term
    – asdf
    Apr 15 at 16:33






  • 3




    It has been proved multiple times on MSE that $$frac{1}{4^n}binom{2n}{n}simfrac{1}{sqrt{pi n}},$$ so... (you do not need Stirling's inequality/approximation to prove it)
    – Jack D'Aurizio
    Apr 15 at 16:33












  • Wait - do you really ask for the series $sum 4^n/{2nchoose n}$ or just for the sequence ${4^n/{2nchoose n}}_{n=0}^infty$?
    – Hagen von Eitzen
    Apr 15 at 16:43












  • I am asking for series.
    – Nebeski
    Apr 15 at 16:47












  • More methods at math.stackexchange.com/questions/1606836/…
    – BAYMAX
    Apr 15 at 17:17


















  • Use Stirling's approximation for the factorial term
    – asdf
    Apr 15 at 16:33






  • 3




    It has been proved multiple times on MSE that $$frac{1}{4^n}binom{2n}{n}simfrac{1}{sqrt{pi n}},$$ so... (you do not need Stirling's inequality/approximation to prove it)
    – Jack D'Aurizio
    Apr 15 at 16:33












  • Wait - do you really ask for the series $sum 4^n/{2nchoose n}$ or just for the sequence ${4^n/{2nchoose n}}_{n=0}^infty$?
    – Hagen von Eitzen
    Apr 15 at 16:43












  • I am asking for series.
    – Nebeski
    Apr 15 at 16:47












  • More methods at math.stackexchange.com/questions/1606836/…
    – BAYMAX
    Apr 15 at 17:17
















Use Stirling's approximation for the factorial term
– asdf
Apr 15 at 16:33




Use Stirling's approximation for the factorial term
– asdf
Apr 15 at 16:33




3




3




It has been proved multiple times on MSE that $$frac{1}{4^n}binom{2n}{n}simfrac{1}{sqrt{pi n}},$$ so... (you do not need Stirling's inequality/approximation to prove it)
– Jack D'Aurizio
Apr 15 at 16:33






It has been proved multiple times on MSE that $$frac{1}{4^n}binom{2n}{n}simfrac{1}{sqrt{pi n}},$$ so... (you do not need Stirling's inequality/approximation to prove it)
– Jack D'Aurizio
Apr 15 at 16:33














Wait - do you really ask for the series $sum 4^n/{2nchoose n}$ or just for the sequence ${4^n/{2nchoose n}}_{n=0}^infty$?
– Hagen von Eitzen
Apr 15 at 16:43






Wait - do you really ask for the series $sum 4^n/{2nchoose n}$ or just for the sequence ${4^n/{2nchoose n}}_{n=0}^infty$?
– Hagen von Eitzen
Apr 15 at 16:43














I am asking for series.
– Nebeski
Apr 15 at 16:47






I am asking for series.
– Nebeski
Apr 15 at 16:47














More methods at math.stackexchange.com/questions/1606836/…
– BAYMAX
Apr 15 at 17:17




More methods at math.stackexchange.com/questions/1606836/…
– BAYMAX
Apr 15 at 17:17










3 Answers
3






active

oldest

votes

















up vote
2
down vote



accepted










Let $a_n:=4^ncdotfrac{(n!)^2}{(2n)!}$, and note that $2^ncdot n!=2cdot4cdot6cdots 2n$. Hence



$$a_n=4^n frac{(n!)^2}{(2n)!}=prod_{k=1}^{n}frac{2k}{2k-1}=expleft(sum_{k=1}^nlogleft(frac{2k}{2k-1}right)right)ge expleft(nlogleft(frac{2n}{2n-1}right)right)tag1$$



and



$$lim_{xtoinfty}xlogleft(frac{2x}{2x-1}right)=lim_{xtoinfty}frac{frac{2(2x-1)-4x}{(2x-1)^2}}{-left(frac{2x}{2x-1}right)frac1{x^2}}=lim_{xtoinfty}frac{2x^2}{(2x-1)2x}=frac12tag2$$



Thus $lim a_nge sqrt e$, so $sum a_n=infty$.





A more simple approach is expanding and chopping the product on $(1)$, that is



$$prod_{k=1}^nfrac{2k}{2k-1}=prod_{k=1}^nleft(1+frac1{2k-1}right)=1+sum_{k=1}^nfrac1{2k-1}+ldotstag3$$



Then we have the lower bound



$$a_nge 1+sum_{k=1}^nfrac1{2k-1}=1+frac12sum_{k=1}^nfrac1{k-1/2}ge 1+frac12sum_{k=1}^nfrac1ktag4$$



Hence $lim a_n=infty$ (because $sumfrac1k=infty$).






share|cite|improve this answer























  • Where does the exponent n disappear in your note?
    – Nebeski
    Apr 15 at 17:17










  • I wasn't talking about that. $2^n×n!=2×4×dots×2n$ only if exponent n=1
    – Nebeski
    Apr 15 at 17:24












  • I mean $2^n×n! = (2^n)×(2×2^n)×dots×(n×2^n)$
    – Nebeski
    Apr 15 at 17:35










  • @Nebeski this $2^n×n! neq (2^n)×(2×2^n)×dots×(n×2^n)$. In the RHS you are multiplying by $2^n$ many times, and in LHS only once.
    – Masacroso
    Apr 15 at 17:38












  • Ohhh yeah, I think all these $×$ confused me.
    – Nebeski
    Apr 15 at 17:45


















up vote
2
down vote













Stirling's approximation? Naah, let us go for a greater overkill. Since the series
$$ sum_{ngeq 0}left[frac{1}{4^n}binom{2n}{n}right]^3 $$
is convergent to $frac{pi}{Gammaleft(frac{3}{4}right)^4}$ due to the relation with the squared complete elliptic integral of the first kind (identity $(7)$ at $k=frac{1}{sqrt{2}}$), its main term is convergent to zero and your sequence is divergent.





Seriously, an elementary approach. Since $2costheta=e^{itheta}+e^{-itheta}$ and $int_{0}^{2pi}e^{nitheta}e^{-mitheta},dtheta = 2pidelta(m,n)$,
$$ frac{1}{4^n}binom{2n}{n}=frac{2}{pi}int_{0}^{pi/2}cos^{2n}theta,dtheta. $$
By the dominated/monotone convergence theorem, the limit of both sides as $nto +infty$ is zero, hence your sequence is divergent. We also have that $left{frac{1}{4^n}binom{2n}{n}right}_{ngeq 1}$ is log-convex due to the Cauchy-Schwarz inequality and the previous integral representation.





Yet another elementary approach. You may prove in a combinatorial fashion that
$$ sum_{k=0}^{n}binom{2k}{k}binom{2n-2k}{n-k} = 4^n tag{Convolution}$$
hence it follows that
$$ left[sum_{k=0}^{n}frac{1}{4^k}binom{2k}{k}right]^2 leq sum_{k=0}^{2n} 1 = 2n $$
and
$$ sum_{k=0}^{n}frac{1}{4^k}binom{2k}{k} leq sqrt{2n}. $$
On the other hand we also have



$$ left[sum_{k=0}^{n}frac{1}{4^k}binom{2k}{k}right]^2 geq sum_{k=0}^{n} 1 = n $$
hence
$$ sqrt{n}leq sum_{k=0}^{n}frac{1}{4^k}binom{2k}{k} leq sqrt{2n}.tag{SumInequality} $$
Since the sequence $left{frac{1}{4^n}binom{2n}{n}right}_{ngeq 1}$ is decreasing, the previous inequality implies that $frac{1}{4^n}binom{2n}{n}llfrac{1}{sqrt{n}}$ as $nto +infty$. The correct asymptotic behaviour is given by Wallis' product and it is
$$ frac{1}{4^n}binom{2n}{n}simfrac{1}{sqrt{pi n}}.tag{GoodToKnow}$$






share|cite|improve this answer



















  • 1




    Thanks for the "elementary" approach too! Stirling is very brutal and overkill but it works so well :)
    – gimusi
    Apr 15 at 20:12


















up vote
1
down vote













By Stirling's approximation



$$n! sim sqrt{2 pi n}left(frac{n}{e}right)^n$$



we have



$$4^n frac{(n!)^2}{(2n)!}sim 4^n frac{2 pi nleft(frac{n}{e}right)^{2n}}{sqrt{4 pi n}left(frac{2n}{e}right)^{2n}}=sqrt{pi n}to infty$$



then the series diverges.






share|cite|improve this answer























    Your Answer





    StackExchange.ifUsing("editor", function () {
    return StackExchange.using("mathjaxEditing", function () {
    StackExchange.MarkdownEditor.creationCallbacks.add(function (editor, postfix) {
    StackExchange.mathjaxEditing.prepareWmdForMathJax(editor, postfix, [["$", "$"], ["\\(","\\)"]]);
    });
    });
    }, "mathjax-editing");

    StackExchange.ready(function() {
    var channelOptions = {
    tags: "".split(" "),
    id: "69"
    };
    initTagRenderer("".split(" "), "".split(" "), channelOptions);

    StackExchange.using("externalEditor", function() {
    // Have to fire editor after snippets, if snippets enabled
    if (StackExchange.settings.snippets.snippetsEnabled) {
    StackExchange.using("snippets", function() {
    createEditor();
    });
    }
    else {
    createEditor();
    }
    });

    function createEditor() {
    StackExchange.prepareEditor({
    heartbeatType: 'answer',
    convertImagesToLinks: true,
    noModals: true,
    showLowRepImageUploadWarning: true,
    reputationToPostImages: 10,
    bindNavPrevention: true,
    postfix: "",
    imageUploader: {
    brandingHtml: "Powered by u003ca class="icon-imgur-white" href="https://imgur.com/"u003eu003c/au003e",
    contentPolicyHtml: "User contributions licensed under u003ca href="https://creativecommons.org/licenses/by-sa/3.0/"u003ecc by-sa 3.0 with attribution requiredu003c/au003e u003ca href="https://stackoverflow.com/legal/content-policy"u003e(content policy)u003c/au003e",
    allowUrls: true
    },
    noCode: true, onDemand: true,
    discardSelector: ".discard-answer"
    ,immediatelyShowMarkdownHelp:true
    });


    }
    });














    draft saved

    draft discarded


















    StackExchange.ready(
    function () {
    StackExchange.openid.initPostLogin('.new-post-login', 'https%3a%2f%2fmath.stackexchange.com%2fquestions%2f2738541%2fdoes-series-4n%25c3%2597-fracn22n-converge-or-diverge%23new-answer', 'question_page');
    }
    );

    Post as a guest















    Required, but never shown

























    3 Answers
    3






    active

    oldest

    votes








    3 Answers
    3






    active

    oldest

    votes









    active

    oldest

    votes






    active

    oldest

    votes








    up vote
    2
    down vote



    accepted










    Let $a_n:=4^ncdotfrac{(n!)^2}{(2n)!}$, and note that $2^ncdot n!=2cdot4cdot6cdots 2n$. Hence



    $$a_n=4^n frac{(n!)^2}{(2n)!}=prod_{k=1}^{n}frac{2k}{2k-1}=expleft(sum_{k=1}^nlogleft(frac{2k}{2k-1}right)right)ge expleft(nlogleft(frac{2n}{2n-1}right)right)tag1$$



    and



    $$lim_{xtoinfty}xlogleft(frac{2x}{2x-1}right)=lim_{xtoinfty}frac{frac{2(2x-1)-4x}{(2x-1)^2}}{-left(frac{2x}{2x-1}right)frac1{x^2}}=lim_{xtoinfty}frac{2x^2}{(2x-1)2x}=frac12tag2$$



    Thus $lim a_nge sqrt e$, so $sum a_n=infty$.





    A more simple approach is expanding and chopping the product on $(1)$, that is



    $$prod_{k=1}^nfrac{2k}{2k-1}=prod_{k=1}^nleft(1+frac1{2k-1}right)=1+sum_{k=1}^nfrac1{2k-1}+ldotstag3$$



    Then we have the lower bound



    $$a_nge 1+sum_{k=1}^nfrac1{2k-1}=1+frac12sum_{k=1}^nfrac1{k-1/2}ge 1+frac12sum_{k=1}^nfrac1ktag4$$



    Hence $lim a_n=infty$ (because $sumfrac1k=infty$).






    share|cite|improve this answer























    • Where does the exponent n disappear in your note?
      – Nebeski
      Apr 15 at 17:17










    • I wasn't talking about that. $2^n×n!=2×4×dots×2n$ only if exponent n=1
      – Nebeski
      Apr 15 at 17:24












    • I mean $2^n×n! = (2^n)×(2×2^n)×dots×(n×2^n)$
      – Nebeski
      Apr 15 at 17:35










    • @Nebeski this $2^n×n! neq (2^n)×(2×2^n)×dots×(n×2^n)$. In the RHS you are multiplying by $2^n$ many times, and in LHS only once.
      – Masacroso
      Apr 15 at 17:38












    • Ohhh yeah, I think all these $×$ confused me.
      – Nebeski
      Apr 15 at 17:45















    up vote
    2
    down vote



    accepted










    Let $a_n:=4^ncdotfrac{(n!)^2}{(2n)!}$, and note that $2^ncdot n!=2cdot4cdot6cdots 2n$. Hence



    $$a_n=4^n frac{(n!)^2}{(2n)!}=prod_{k=1}^{n}frac{2k}{2k-1}=expleft(sum_{k=1}^nlogleft(frac{2k}{2k-1}right)right)ge expleft(nlogleft(frac{2n}{2n-1}right)right)tag1$$



    and



    $$lim_{xtoinfty}xlogleft(frac{2x}{2x-1}right)=lim_{xtoinfty}frac{frac{2(2x-1)-4x}{(2x-1)^2}}{-left(frac{2x}{2x-1}right)frac1{x^2}}=lim_{xtoinfty}frac{2x^2}{(2x-1)2x}=frac12tag2$$



    Thus $lim a_nge sqrt e$, so $sum a_n=infty$.





    A more simple approach is expanding and chopping the product on $(1)$, that is



    $$prod_{k=1}^nfrac{2k}{2k-1}=prod_{k=1}^nleft(1+frac1{2k-1}right)=1+sum_{k=1}^nfrac1{2k-1}+ldotstag3$$



    Then we have the lower bound



    $$a_nge 1+sum_{k=1}^nfrac1{2k-1}=1+frac12sum_{k=1}^nfrac1{k-1/2}ge 1+frac12sum_{k=1}^nfrac1ktag4$$



    Hence $lim a_n=infty$ (because $sumfrac1k=infty$).






    share|cite|improve this answer























    • Where does the exponent n disappear in your note?
      – Nebeski
      Apr 15 at 17:17










    • I wasn't talking about that. $2^n×n!=2×4×dots×2n$ only if exponent n=1
      – Nebeski
      Apr 15 at 17:24












    • I mean $2^n×n! = (2^n)×(2×2^n)×dots×(n×2^n)$
      – Nebeski
      Apr 15 at 17:35










    • @Nebeski this $2^n×n! neq (2^n)×(2×2^n)×dots×(n×2^n)$. In the RHS you are multiplying by $2^n$ many times, and in LHS only once.
      – Masacroso
      Apr 15 at 17:38












    • Ohhh yeah, I think all these $×$ confused me.
      – Nebeski
      Apr 15 at 17:45













    up vote
    2
    down vote



    accepted







    up vote
    2
    down vote



    accepted






    Let $a_n:=4^ncdotfrac{(n!)^2}{(2n)!}$, and note that $2^ncdot n!=2cdot4cdot6cdots 2n$. Hence



    $$a_n=4^n frac{(n!)^2}{(2n)!}=prod_{k=1}^{n}frac{2k}{2k-1}=expleft(sum_{k=1}^nlogleft(frac{2k}{2k-1}right)right)ge expleft(nlogleft(frac{2n}{2n-1}right)right)tag1$$



    and



    $$lim_{xtoinfty}xlogleft(frac{2x}{2x-1}right)=lim_{xtoinfty}frac{frac{2(2x-1)-4x}{(2x-1)^2}}{-left(frac{2x}{2x-1}right)frac1{x^2}}=lim_{xtoinfty}frac{2x^2}{(2x-1)2x}=frac12tag2$$



    Thus $lim a_nge sqrt e$, so $sum a_n=infty$.





    A more simple approach is expanding and chopping the product on $(1)$, that is



    $$prod_{k=1}^nfrac{2k}{2k-1}=prod_{k=1}^nleft(1+frac1{2k-1}right)=1+sum_{k=1}^nfrac1{2k-1}+ldotstag3$$



    Then we have the lower bound



    $$a_nge 1+sum_{k=1}^nfrac1{2k-1}=1+frac12sum_{k=1}^nfrac1{k-1/2}ge 1+frac12sum_{k=1}^nfrac1ktag4$$



    Hence $lim a_n=infty$ (because $sumfrac1k=infty$).






    share|cite|improve this answer














    Let $a_n:=4^ncdotfrac{(n!)^2}{(2n)!}$, and note that $2^ncdot n!=2cdot4cdot6cdots 2n$. Hence



    $$a_n=4^n frac{(n!)^2}{(2n)!}=prod_{k=1}^{n}frac{2k}{2k-1}=expleft(sum_{k=1}^nlogleft(frac{2k}{2k-1}right)right)ge expleft(nlogleft(frac{2n}{2n-1}right)right)tag1$$



    and



    $$lim_{xtoinfty}xlogleft(frac{2x}{2x-1}right)=lim_{xtoinfty}frac{frac{2(2x-1)-4x}{(2x-1)^2}}{-left(frac{2x}{2x-1}right)frac1{x^2}}=lim_{xtoinfty}frac{2x^2}{(2x-1)2x}=frac12tag2$$



    Thus $lim a_nge sqrt e$, so $sum a_n=infty$.





    A more simple approach is expanding and chopping the product on $(1)$, that is



    $$prod_{k=1}^nfrac{2k}{2k-1}=prod_{k=1}^nleft(1+frac1{2k-1}right)=1+sum_{k=1}^nfrac1{2k-1}+ldotstag3$$



    Then we have the lower bound



    $$a_nge 1+sum_{k=1}^nfrac1{2k-1}=1+frac12sum_{k=1}^nfrac1{k-1/2}ge 1+frac12sum_{k=1}^nfrac1ktag4$$



    Hence $lim a_n=infty$ (because $sumfrac1k=infty$).







    share|cite|improve this answer














    share|cite|improve this answer



    share|cite|improve this answer








    edited Apr 15 at 18:10

























    answered Apr 15 at 16:46









    Masacroso

    12.3k41746




    12.3k41746












    • Where does the exponent n disappear in your note?
      – Nebeski
      Apr 15 at 17:17










    • I wasn't talking about that. $2^n×n!=2×4×dots×2n$ only if exponent n=1
      – Nebeski
      Apr 15 at 17:24












    • I mean $2^n×n! = (2^n)×(2×2^n)×dots×(n×2^n)$
      – Nebeski
      Apr 15 at 17:35










    • @Nebeski this $2^n×n! neq (2^n)×(2×2^n)×dots×(n×2^n)$. In the RHS you are multiplying by $2^n$ many times, and in LHS only once.
      – Masacroso
      Apr 15 at 17:38












    • Ohhh yeah, I think all these $×$ confused me.
      – Nebeski
      Apr 15 at 17:45


















    • Where does the exponent n disappear in your note?
      – Nebeski
      Apr 15 at 17:17










    • I wasn't talking about that. $2^n×n!=2×4×dots×2n$ only if exponent n=1
      – Nebeski
      Apr 15 at 17:24












    • I mean $2^n×n! = (2^n)×(2×2^n)×dots×(n×2^n)$
      – Nebeski
      Apr 15 at 17:35










    • @Nebeski this $2^n×n! neq (2^n)×(2×2^n)×dots×(n×2^n)$. In the RHS you are multiplying by $2^n$ many times, and in LHS only once.
      – Masacroso
      Apr 15 at 17:38












    • Ohhh yeah, I think all these $×$ confused me.
      – Nebeski
      Apr 15 at 17:45
















    Where does the exponent n disappear in your note?
    – Nebeski
    Apr 15 at 17:17




    Where does the exponent n disappear in your note?
    – Nebeski
    Apr 15 at 17:17












    I wasn't talking about that. $2^n×n!=2×4×dots×2n$ only if exponent n=1
    – Nebeski
    Apr 15 at 17:24






    I wasn't talking about that. $2^n×n!=2×4×dots×2n$ only if exponent n=1
    – Nebeski
    Apr 15 at 17:24














    I mean $2^n×n! = (2^n)×(2×2^n)×dots×(n×2^n)$
    – Nebeski
    Apr 15 at 17:35




    I mean $2^n×n! = (2^n)×(2×2^n)×dots×(n×2^n)$
    – Nebeski
    Apr 15 at 17:35












    @Nebeski this $2^n×n! neq (2^n)×(2×2^n)×dots×(n×2^n)$. In the RHS you are multiplying by $2^n$ many times, and in LHS only once.
    – Masacroso
    Apr 15 at 17:38






    @Nebeski this $2^n×n! neq (2^n)×(2×2^n)×dots×(n×2^n)$. In the RHS you are multiplying by $2^n$ many times, and in LHS only once.
    – Masacroso
    Apr 15 at 17:38














    Ohhh yeah, I think all these $×$ confused me.
    – Nebeski
    Apr 15 at 17:45




    Ohhh yeah, I think all these $×$ confused me.
    – Nebeski
    Apr 15 at 17:45










    up vote
    2
    down vote













    Stirling's approximation? Naah, let us go for a greater overkill. Since the series
    $$ sum_{ngeq 0}left[frac{1}{4^n}binom{2n}{n}right]^3 $$
    is convergent to $frac{pi}{Gammaleft(frac{3}{4}right)^4}$ due to the relation with the squared complete elliptic integral of the first kind (identity $(7)$ at $k=frac{1}{sqrt{2}}$), its main term is convergent to zero and your sequence is divergent.





    Seriously, an elementary approach. Since $2costheta=e^{itheta}+e^{-itheta}$ and $int_{0}^{2pi}e^{nitheta}e^{-mitheta},dtheta = 2pidelta(m,n)$,
    $$ frac{1}{4^n}binom{2n}{n}=frac{2}{pi}int_{0}^{pi/2}cos^{2n}theta,dtheta. $$
    By the dominated/monotone convergence theorem, the limit of both sides as $nto +infty$ is zero, hence your sequence is divergent. We also have that $left{frac{1}{4^n}binom{2n}{n}right}_{ngeq 1}$ is log-convex due to the Cauchy-Schwarz inequality and the previous integral representation.





    Yet another elementary approach. You may prove in a combinatorial fashion that
    $$ sum_{k=0}^{n}binom{2k}{k}binom{2n-2k}{n-k} = 4^n tag{Convolution}$$
    hence it follows that
    $$ left[sum_{k=0}^{n}frac{1}{4^k}binom{2k}{k}right]^2 leq sum_{k=0}^{2n} 1 = 2n $$
    and
    $$ sum_{k=0}^{n}frac{1}{4^k}binom{2k}{k} leq sqrt{2n}. $$
    On the other hand we also have



    $$ left[sum_{k=0}^{n}frac{1}{4^k}binom{2k}{k}right]^2 geq sum_{k=0}^{n} 1 = n $$
    hence
    $$ sqrt{n}leq sum_{k=0}^{n}frac{1}{4^k}binom{2k}{k} leq sqrt{2n}.tag{SumInequality} $$
    Since the sequence $left{frac{1}{4^n}binom{2n}{n}right}_{ngeq 1}$ is decreasing, the previous inequality implies that $frac{1}{4^n}binom{2n}{n}llfrac{1}{sqrt{n}}$ as $nto +infty$. The correct asymptotic behaviour is given by Wallis' product and it is
    $$ frac{1}{4^n}binom{2n}{n}simfrac{1}{sqrt{pi n}}.tag{GoodToKnow}$$






    share|cite|improve this answer



















    • 1




      Thanks for the "elementary" approach too! Stirling is very brutal and overkill but it works so well :)
      – gimusi
      Apr 15 at 20:12















    up vote
    2
    down vote













    Stirling's approximation? Naah, let us go for a greater overkill. Since the series
    $$ sum_{ngeq 0}left[frac{1}{4^n}binom{2n}{n}right]^3 $$
    is convergent to $frac{pi}{Gammaleft(frac{3}{4}right)^4}$ due to the relation with the squared complete elliptic integral of the first kind (identity $(7)$ at $k=frac{1}{sqrt{2}}$), its main term is convergent to zero and your sequence is divergent.





    Seriously, an elementary approach. Since $2costheta=e^{itheta}+e^{-itheta}$ and $int_{0}^{2pi}e^{nitheta}e^{-mitheta},dtheta = 2pidelta(m,n)$,
    $$ frac{1}{4^n}binom{2n}{n}=frac{2}{pi}int_{0}^{pi/2}cos^{2n}theta,dtheta. $$
    By the dominated/monotone convergence theorem, the limit of both sides as $nto +infty$ is zero, hence your sequence is divergent. We also have that $left{frac{1}{4^n}binom{2n}{n}right}_{ngeq 1}$ is log-convex due to the Cauchy-Schwarz inequality and the previous integral representation.





    Yet another elementary approach. You may prove in a combinatorial fashion that
    $$ sum_{k=0}^{n}binom{2k}{k}binom{2n-2k}{n-k} = 4^n tag{Convolution}$$
    hence it follows that
    $$ left[sum_{k=0}^{n}frac{1}{4^k}binom{2k}{k}right]^2 leq sum_{k=0}^{2n} 1 = 2n $$
    and
    $$ sum_{k=0}^{n}frac{1}{4^k}binom{2k}{k} leq sqrt{2n}. $$
    On the other hand we also have



    $$ left[sum_{k=0}^{n}frac{1}{4^k}binom{2k}{k}right]^2 geq sum_{k=0}^{n} 1 = n $$
    hence
    $$ sqrt{n}leq sum_{k=0}^{n}frac{1}{4^k}binom{2k}{k} leq sqrt{2n}.tag{SumInequality} $$
    Since the sequence $left{frac{1}{4^n}binom{2n}{n}right}_{ngeq 1}$ is decreasing, the previous inequality implies that $frac{1}{4^n}binom{2n}{n}llfrac{1}{sqrt{n}}$ as $nto +infty$. The correct asymptotic behaviour is given by Wallis' product and it is
    $$ frac{1}{4^n}binom{2n}{n}simfrac{1}{sqrt{pi n}}.tag{GoodToKnow}$$






    share|cite|improve this answer



















    • 1




      Thanks for the "elementary" approach too! Stirling is very brutal and overkill but it works so well :)
      – gimusi
      Apr 15 at 20:12













    up vote
    2
    down vote










    up vote
    2
    down vote









    Stirling's approximation? Naah, let us go for a greater overkill. Since the series
    $$ sum_{ngeq 0}left[frac{1}{4^n}binom{2n}{n}right]^3 $$
    is convergent to $frac{pi}{Gammaleft(frac{3}{4}right)^4}$ due to the relation with the squared complete elliptic integral of the first kind (identity $(7)$ at $k=frac{1}{sqrt{2}}$), its main term is convergent to zero and your sequence is divergent.





    Seriously, an elementary approach. Since $2costheta=e^{itheta}+e^{-itheta}$ and $int_{0}^{2pi}e^{nitheta}e^{-mitheta},dtheta = 2pidelta(m,n)$,
    $$ frac{1}{4^n}binom{2n}{n}=frac{2}{pi}int_{0}^{pi/2}cos^{2n}theta,dtheta. $$
    By the dominated/monotone convergence theorem, the limit of both sides as $nto +infty$ is zero, hence your sequence is divergent. We also have that $left{frac{1}{4^n}binom{2n}{n}right}_{ngeq 1}$ is log-convex due to the Cauchy-Schwarz inequality and the previous integral representation.





    Yet another elementary approach. You may prove in a combinatorial fashion that
    $$ sum_{k=0}^{n}binom{2k}{k}binom{2n-2k}{n-k} = 4^n tag{Convolution}$$
    hence it follows that
    $$ left[sum_{k=0}^{n}frac{1}{4^k}binom{2k}{k}right]^2 leq sum_{k=0}^{2n} 1 = 2n $$
    and
    $$ sum_{k=0}^{n}frac{1}{4^k}binom{2k}{k} leq sqrt{2n}. $$
    On the other hand we also have



    $$ left[sum_{k=0}^{n}frac{1}{4^k}binom{2k}{k}right]^2 geq sum_{k=0}^{n} 1 = n $$
    hence
    $$ sqrt{n}leq sum_{k=0}^{n}frac{1}{4^k}binom{2k}{k} leq sqrt{2n}.tag{SumInequality} $$
    Since the sequence $left{frac{1}{4^n}binom{2n}{n}right}_{ngeq 1}$ is decreasing, the previous inequality implies that $frac{1}{4^n}binom{2n}{n}llfrac{1}{sqrt{n}}$ as $nto +infty$. The correct asymptotic behaviour is given by Wallis' product and it is
    $$ frac{1}{4^n}binom{2n}{n}simfrac{1}{sqrt{pi n}}.tag{GoodToKnow}$$






    share|cite|improve this answer














    Stirling's approximation? Naah, let us go for a greater overkill. Since the series
    $$ sum_{ngeq 0}left[frac{1}{4^n}binom{2n}{n}right]^3 $$
    is convergent to $frac{pi}{Gammaleft(frac{3}{4}right)^4}$ due to the relation with the squared complete elliptic integral of the first kind (identity $(7)$ at $k=frac{1}{sqrt{2}}$), its main term is convergent to zero and your sequence is divergent.





    Seriously, an elementary approach. Since $2costheta=e^{itheta}+e^{-itheta}$ and $int_{0}^{2pi}e^{nitheta}e^{-mitheta},dtheta = 2pidelta(m,n)$,
    $$ frac{1}{4^n}binom{2n}{n}=frac{2}{pi}int_{0}^{pi/2}cos^{2n}theta,dtheta. $$
    By the dominated/monotone convergence theorem, the limit of both sides as $nto +infty$ is zero, hence your sequence is divergent. We also have that $left{frac{1}{4^n}binom{2n}{n}right}_{ngeq 1}$ is log-convex due to the Cauchy-Schwarz inequality and the previous integral representation.





    Yet another elementary approach. You may prove in a combinatorial fashion that
    $$ sum_{k=0}^{n}binom{2k}{k}binom{2n-2k}{n-k} = 4^n tag{Convolution}$$
    hence it follows that
    $$ left[sum_{k=0}^{n}frac{1}{4^k}binom{2k}{k}right]^2 leq sum_{k=0}^{2n} 1 = 2n $$
    and
    $$ sum_{k=0}^{n}frac{1}{4^k}binom{2k}{k} leq sqrt{2n}. $$
    On the other hand we also have



    $$ left[sum_{k=0}^{n}frac{1}{4^k}binom{2k}{k}right]^2 geq sum_{k=0}^{n} 1 = n $$
    hence
    $$ sqrt{n}leq sum_{k=0}^{n}frac{1}{4^k}binom{2k}{k} leq sqrt{2n}.tag{SumInequality} $$
    Since the sequence $left{frac{1}{4^n}binom{2n}{n}right}_{ngeq 1}$ is decreasing, the previous inequality implies that $frac{1}{4^n}binom{2n}{n}llfrac{1}{sqrt{n}}$ as $nto +infty$. The correct asymptotic behaviour is given by Wallis' product and it is
    $$ frac{1}{4^n}binom{2n}{n}simfrac{1}{sqrt{pi n}}.tag{GoodToKnow}$$







    share|cite|improve this answer














    share|cite|improve this answer



    share|cite|improve this answer








    edited Apr 15 at 20:17

























    answered Apr 15 at 19:57









    Jack D'Aurizio

    284k33275653




    284k33275653








    • 1




      Thanks for the "elementary" approach too! Stirling is very brutal and overkill but it works so well :)
      – gimusi
      Apr 15 at 20:12














    • 1




      Thanks for the "elementary" approach too! Stirling is very brutal and overkill but it works so well :)
      – gimusi
      Apr 15 at 20:12








    1




    1




    Thanks for the "elementary" approach too! Stirling is very brutal and overkill but it works so well :)
    – gimusi
    Apr 15 at 20:12




    Thanks for the "elementary" approach too! Stirling is very brutal and overkill but it works so well :)
    – gimusi
    Apr 15 at 20:12










    up vote
    1
    down vote













    By Stirling's approximation



    $$n! sim sqrt{2 pi n}left(frac{n}{e}right)^n$$



    we have



    $$4^n frac{(n!)^2}{(2n)!}sim 4^n frac{2 pi nleft(frac{n}{e}right)^{2n}}{sqrt{4 pi n}left(frac{2n}{e}right)^{2n}}=sqrt{pi n}to infty$$



    then the series diverges.






    share|cite|improve this answer



























      up vote
      1
      down vote













      By Stirling's approximation



      $$n! sim sqrt{2 pi n}left(frac{n}{e}right)^n$$



      we have



      $$4^n frac{(n!)^2}{(2n)!}sim 4^n frac{2 pi nleft(frac{n}{e}right)^{2n}}{sqrt{4 pi n}left(frac{2n}{e}right)^{2n}}=sqrt{pi n}to infty$$



      then the series diverges.






      share|cite|improve this answer

























        up vote
        1
        down vote










        up vote
        1
        down vote









        By Stirling's approximation



        $$n! sim sqrt{2 pi n}left(frac{n}{e}right)^n$$



        we have



        $$4^n frac{(n!)^2}{(2n)!}sim 4^n frac{2 pi nleft(frac{n}{e}right)^{2n}}{sqrt{4 pi n}left(frac{2n}{e}right)^{2n}}=sqrt{pi n}to infty$$



        then the series diverges.






        share|cite|improve this answer














        By Stirling's approximation



        $$n! sim sqrt{2 pi n}left(frac{n}{e}right)^n$$



        we have



        $$4^n frac{(n!)^2}{(2n)!}sim 4^n frac{2 pi nleft(frac{n}{e}right)^{2n}}{sqrt{4 pi n}left(frac{2n}{e}right)^{2n}}=sqrt{pi n}to infty$$



        then the series diverges.







        share|cite|improve this answer














        share|cite|improve this answer



        share|cite|improve this answer








        edited Apr 15 at 19:55









        Masacroso

        12.3k41746




        12.3k41746










        answered Apr 15 at 17:12









        gimusi

        89.3k74495




        89.3k74495






























            draft saved

            draft discarded




















































            Thanks for contributing an answer to Mathematics Stack Exchange!


            • Please be sure to answer the question. Provide details and share your research!

            But avoid



            • Asking for help, clarification, or responding to other answers.

            • Making statements based on opinion; back them up with references or personal experience.


            Use MathJax to format equations. MathJax reference.


            To learn more, see our tips on writing great answers.





            Some of your past answers have not been well-received, and you're in danger of being blocked from answering.


            Please pay close attention to the following guidance:


            • Please be sure to answer the question. Provide details and share your research!

            But avoid



            • Asking for help, clarification, or responding to other answers.

            • Making statements based on opinion; back them up with references or personal experience.


            To learn more, see our tips on writing great answers.




            draft saved


            draft discarded














            StackExchange.ready(
            function () {
            StackExchange.openid.initPostLogin('.new-post-login', 'https%3a%2f%2fmath.stackexchange.com%2fquestions%2f2738541%2fdoes-series-4n%25c3%2597-fracn22n-converge-or-diverge%23new-answer', 'question_page');
            }
            );

            Post as a guest















            Required, but never shown





















































            Required, but never shown














            Required, but never shown












            Required, but never shown







            Required, but never shown

































            Required, but never shown














            Required, but never shown












            Required, but never shown







            Required, but never shown







            Popular posts from this blog

            QoS: MAC-Priority for clients behind a repeater

            Ивакино (Тотемский район)

            Can't locate Autom4te/ChannelDefs.pm in @INC (when it definitely is there)